What is the min value

Algebra Level 3

Real numbers x x , y y , and z z are such that x + y + z = 1 x+y+z=1 . Find the minimum value of

x 2 1 + x 2 + y 2 1 + y 2 + z 2 1 + z 2 \frac {x^2}{1+x^2} + \frac {y^2}{1+y^2}+ \frac {z^2}{1+z^2}


The answer is 0.3.

This section requires Javascript.
You are seeing this because something didn't load right. We suggest you, (a) try refreshing the page, (b) enabling javascript if it is disabled on your browser and, finally, (c) loading the non-javascript version of this page . We're sorry about the hassle.

2 solutions

Differenting the given constraint we get d x + d y + d z = 0 dx+dy+dz=0

For f ( x , y , z ) = x 2 1 + x 2 + y 2 1 + y 2 + z 2 1 + z 2 f(x,y,z)=\dfrac {x^2}{1+x^2}+\dfrac {y^2}{1+y^2}+\dfrac {z^2}{1+z^2} to be maximum,

2 x ( 1 + x 2 ) 2 d x + 2 y ( 1 + y 2 ) 2 d y + 2 z ( 1 + z 2 ) 2 d z = 0 \dfrac {2x}{(1+x^2)^2}dx+\dfrac {2y}{(1+y^2)^2}dy+\dfrac {2z}{(1+z^2)^2}dz=0 .

Applying LaGrange's method of undetermined multipliers and using the constraint x + y + z = 1 x+y+z=1 , we get x = y = z = 1 3 x=y=z=\dfrac {1}{3}

The maximum value of the given expression is 3 × 1 9 1 + 1 9 = 3 10 = 0.3 3\times \dfrac {\frac{1}{9}}{1+\frac{1}{9}}=\dfrac {3}{10}=\boxed {0.3} .

Solution to this problem :

One obvious number for which n + n 64 \sqrt n+\sqrt {n-64} is an integer is 64 64 .

For other values, let n = m 2 n=m^2 , where m m is an integer. Then

m 2 64 m^2-64 must be a perfect square, say, p 2 p^2 , where p p is an integer. So p , 8 , m p, 8,m form a Pythagorean triplet. The primitive one is 8 , 15 , 17 8,15,17 , while the nonprimitive one is 6 , 8 , 10 6,8,10 . So there are two other values of n : 1 7 2 = 289 n: 17^2=289 and n = 1 0 2 = 100 n=10^2=100 .

Therefore in all, there are 3 \boxed 3 values of n n satisfying the given condition.

Chew-Seong Cheong
Jul 12, 2020

We note that f ( x ) = x 2 1 + x 2 f(x) = \dfrac {x^2}{1+x^2} is convex within x [ 0 , 1 ] x \in [0,1] . By Jensen's inequality ,

1 3 ( x 2 1 + x 2 + y 2 1 + y 2 + z 2 1 + z 2 ) ( x + y + z 3 ) 2 1 + ( x + y + z 3 ) 2 = 1 9 1 + 1 9 = 1 10 x 2 1 + x 2 + y 2 1 + y 2 + z 2 1 + z 2 3 10 = 0.3 \begin{aligned} \frac 13 \left(\frac {x^2}{1+x^2} + \frac {y^2}{1+y^2} + \frac {z^2}{1+z^2} \right) & \ge \frac {\left(\frac {x+y+z}3\right)^2}{1+ \left(\frac {x+y+z}3\right)^2} = \frac {\frac 19}{1+\frac 19} = \frac 1{10} \\ \implies \frac {x^2}{1+x^2} + \frac {y^2}{1+y^2} + \frac {z^2}{1+z^2} & \ge \frac 3{10} = \boxed{0.3} \end{aligned}

0 pending reports

×

Problem Loading...

Note Loading...

Set Loading...